PT70.S1.Q13 - the television network's advertisement

Tina ChoTina Cho Free Trial Member
edited August 2016 in Logical Reasoning 442 karma
https://7sage.com/lsat_explanations/lsat-70-section-1-question-13/
I thought A is the correct answer because it sounds if most viewers started to watch the 1st episode of the program because of the network's ad, it sounds the ad had huge influence or better influence. After all "most" means more than 50%. (Nothing can be more influential than this because it's more than 50%) Why is A wrong? :(

Comments

  • Tina ChoTina Cho Free Trial Member
    442 karma
    wait..does A weaken the argument...?
  • danielznelsondanielznelson Alum Inactive Sage Inactive ⭐
    4181 karma
    This is a very tricky question, especially because it's so early in the test!

    The argument is claiming that the network's ad will not as effectively attract viewers likely to continue watching the program, as they will be unlikely to watch subsequent episodes, because of the misrepresentation. We don't care really about the ad's influence on people's initial choice to check out the show; we care only about the ad's effect on the chances of people watching subsequent episodes.

    "A" states that most viewers who actually do tune to the first episode in will do so because of the ad. But think about what "A" is stating and where it gets us. What does this do for the claim that these viewers won't come back, and more importantly, why the producer's ad would be more likely to help keep these viewers? Even if all viewers tuned in to the first episode, we still know nothing of the likelihood of them sticking around.

    If "A" is negated, it would read "Most viewers who tune in to the first episode of the program will not do so because of the network's advertisement for the program." I mean, that's great and all, but we still don't know the likelihood of those people sticking around. Nothing of the argument is destroyed here, especially the comparative statement that one ad (the producer's) would be better with viewer retention than the other ad (the actual ad).

    Does that help at all?
  • Tina ChoTina Cho Free Trial Member
    442 karma
    Hi daniel, thank you for your explanation, it makes much more sense! :D
    For some reason I thought A could weaken the arg. bc if the ad makes most ppl watch the 1st episode, it could mean the ad is efficient, but as you said the conclusion is about whether they will continue to watch the video (and thats the criteria to judge whether ad is efficient) so A is irrelevant maybe...?

    Thanks again!
  • mudiversmudivers Alum Member
    89 karma
    Conclusion: "It will not as effectively attract...viewers...as would the advertisements...producers favored"

    The conclusion is a comparative statement.

    Premise: TV advertisement grossly misrepresents the program.
    Premise: People who watch...will be unlikely to watch subsequent episodes

    How can we say that the program producer's advertisement would have been more effective? The only way that is possible is if their favored advertisement didn't "grossly misrepresent" (B).

    Daniel's negation will also show that (A) doesn't impact the argument. We simply don't care why they tune into the first episode.

Sign In or Register to comment.